Prove Function is Constant on [a,b]

  • Thread starter Thread starter chief12
  • Start date Start date
  • Tags Tags
    Constant Function
Click For Summary
The discussion revolves around proving that a continuous function f: [a,b] → Q is constant on the interval [a,b]. A proof by contradiction is suggested, starting with the assumption that f is not constant, leading to the existence of two points c and d in [a,b] where f(c) is not equal to f(d). The intermediate value theorem is referenced, highlighting that between any two rational outputs, there must be an irrational number, which contradicts the function's mapping to rational numbers only. The conversation emphasizes the need for a solid understanding of the intermediate value theorem to complete the proof. Ultimately, the conclusion is that if f is continuous and maps to the rationals, it must be constant on the interval.
chief12
Messages
9
Reaction score
0

Homework Statement



for function f:[a,b]--> Q is continuous on [a,b],
prove that f is constant on [a,b]


Homework Equations





The Attempt at a Solution



proof by contradiction,
suppose there is a k, such that a<k<b,
since f is continuous on [a,b], there must be a j, such that f(k)=j.

Since on any fixed interval there is an uncountable number of irrational numbers, then there must be a k, such that f(k) = j and j is not a rational number.

therefore is f:[a,b] --> Q is continuous, f must be constant on [a,b]



I was told that my suggestion is "plausible" but not a proof. Any help? thanks
 
Physics news on Phys.org
A contradiction proof sounds like the right way to start.

So assume: f:[a,b]--> Q is continuous on [a,b] and that that f is not constant on [a,b].

So there exists c,d in [a,b] such that f(c) != f(d). (WLOG, f(c)<f(d)). Between any two rational numbers there exists an irrational number, so there exists e such that f(c) < e < f(d). Using the intermediate value theorem, what do we know?

Just realized, do you "know" intermediate value theorem yet? If not you might have to take another approach.
 
Last edited:
The mean value theorem won't help. It isn't given that the function is differentiable.
 
Stephen Tashi said:
The mean value theorem won't help. It isn't given that the function is differentiable.

Yeah, I fixed it. Was getting them confused.
 
Question: A clock's minute hand has length 4 and its hour hand has length 3. What is the distance between the tips at the moment when it is increasing most rapidly?(Putnam Exam Question) Answer: Making assumption that both the hands moves at constant angular velocities, the answer is ## \sqrt{7} .## But don't you think this assumption is somewhat doubtful and wrong?

Similar threads

  • · Replies 11 ·
Replies
11
Views
2K
  • · Replies 5 ·
Replies
5
Views
2K
Replies
1
Views
2K
Replies
1
Views
2K
Replies
20
Views
4K
  • · Replies 3 ·
Replies
3
Views
2K
  • · Replies 4 ·
Replies
4
Views
1K
  • · Replies 13 ·
Replies
13
Views
2K
  • · Replies 51 ·
2
Replies
51
Views
5K
  • · Replies 26 ·
Replies
26
Views
3K